Answer:
a) 94.02% probability that the selected item is non-defective
b) 99.04% probability that the machine is set up correctly
Step-by-step explanation:
The Bayes Theorem is important to solve this question.
Bayes Theorem:
Two events, A and B.
\(P(B|A) = \frac{P(B)*P(A|B)}{P(A)}\)
In which P(B|A) is the probability of B happening when A has happened and P(A|B) is the probability of A happening when B has happened.
a. An item from the production line is selected. What is the probability that the selected item is non-defective?
The machine is set up correctly 97% of the time. When it is set up correctly, 96% of the items are non-defective.
The other 100-97 = 3% of the time, the machine is set up incorrectly. Then, 30% of the items are non-defective.
So
P = 0.97*0.96 + 0.03*0.3 = 0.9402
94.02% probability that the selected item is non-defective.
b. Given that the selected item is non-defective, what is the probability that the machine is set up correctly?
Then, for the Bayes Theorem:
Event A: The item is non-defective.
Event B: Machine set up correctly.
94.02% probability that the selected item is non-defective.
This means that \(P(A) = 0.9402\)
97% of the time the machine is set up correctly.
This means that \(P(B) = 0.97\)
Furthermore, it is known that if the machine is set up correctly, it produces 96% acceptable (non-defective) items.
This means that \(P(A|B) = 0.96\)
Probability:
\(P(B|A) = \frac{0.97*0.96}{0.9402} = 0.9904\)
99.04% probability that the machine is set up correctly
1493600÷8 i need full steps
When 1,493,600 is divided by 8 the quotient is 186,700.
To divide 1,493,600 by 8, you can follow these steps:
We have to write down the dividend (1,493,600) and the divisor (8).
Now start with the largest place value in the dividend (the leftmost digit) and perform the division.
Divide 1 by 8. Since 1 is smaller than 8, you move to the next digit.
Bring down the next digit (4) and combine it with the previous quotient (0). This gives you 04.
Divide 4 by 8. Since 4 is smaller than 8, you move to the next digit.
Bring down the next digit (9) and combine it with the previous quotient (0). This gives you 09.
Divide 9 by 8. The quotient is 1, and the remainder is 1.
Bring down the next digit (3) and combine it with the remainder (1). This gives you 13.
Divide 13 by 8. The quotient is 1, and the remainder is 5.
Bring down the next digit (6) and combine it with the remainder (5). This gives you 56.
Divide 56 by 8. The quotient is 7, and there is no remainder.
Bring down the next digit (0) and combine it with the quotient (7). This gives you 70.
Divide 70 by 8. The quotient is 8, and there is no remainder.
There are no more digits to bring down, and the division is complete.
The quotient is the result of the division. In this case, 1,493,600 divided by 8 is equal to 186,700.
Therefore, the result of 1,493,600 ÷ 8 is 186,700.
To learn more on Division click:
https://brainly.com/question/21416852
#SPJ1
Need help with the 3rd one only. Will give brainliest and 5 star rating for fast replies.
Answer:
\(f^{-1}(0)=4\)
Step-by-step explanation:
Given f(4) = 0, you want f^-1(4).
Inverse functionFor function y = f(x), the inverse function gives you the value of x corresponding to a given y-value.
That is, ...
\(f(4)=0\ \Longleftrightarrow\ \boxed{f^{-1}(0)=4}\)
__
Additional comment
An inverse function swaps the roles of x and y. It changes the ordered pair (x1, y1) to the pair (y1, x1). That is, f^-1(y1) = x1.
Here, your second part gives you the answer to the third part.
Insert one term between -13 and -119 to create an arithmetic sequence of three terms.
Answer:
-66
Step-by-step explanation:
- 13, ____, -119
first, find the common difference:
-13 - (-119) = 106
next, divide 106 by 2: \(\frac{106}{2}\) = 53
so, the common difference is 53.
now, subtract 53 from -13:
-13 - 53 = -66
Answer:
-66
Step-by-step explanation:
the sequence goes -13,-66,-119
just subtract with 53
(decreasing arithmetic sequence)
5. In the figure Find X (In the picture) (giving points to best answer/brainlest)
Answer:
x=93°
Step-by-step explanation:
the lower left interior angle of the triangle is 39° because it is an alternate-interior angle to the one marked as 39°
the lower right interior angle is 180-132, which is 87°
therefore, x=93° because the three angles must add to 180°
A textbook store sold a combined total of 259 chemistry and sociology textbooks in a week. The number of sociology textbooks sold was 81 less than the number of chemistry textbooks sold. How many textbooks of each type were sold?
178
259-81 you have to subtract
What are the intercepts of the graphed function?
x-intercept = (–1, 0)
y-intercept = (–3, 0)
x-intercept = (0, –1)
y-intercept = (0, –3)
x-intercept = (0, –1)
y-intercept = (–3, 0)
x-intercept = (–1, 0)
y-intercept = (0, –3)
Using it's concepts, the intercepts of the graphed function are:
x-intercept = (–1, 0).y-intercept = (0, –3).What is the missing information?The graph of the function is missing, and is given at the end of the answer.
What are the x-intercepts of a function?The x-intercepts of a function f(x) are the values of x for which f(x) = 0, that is, the values of x for which the function crosses the x-axis. Looking at the researched graph, there is only one x-intercept at x = -1, hence the point is (-1,0).
What is the y-intercept of a function?The y-intercept of a function f(x) is the value of f(x) for which x = 0, the value of y when the function crosses the y-axis. From the same graph stated above in the x-intercept, when x = 0, y = -3, hence the point is (0, -3).
More can be learned about intercepts at https://brainly.com/question/24363347
#SPJ1
(08.03|08.04 HC)
For the regions A and B shown in the graph:
Part A: Discuss the limits of integration. (3 points)
Part B: Set up an integral expression that represents the total area. (4 points)
Part C: Calculate the total area. (3 points)
The total area from the graph is 2.737.
What is area?Area is the amount of space occupied by a two-dimensional figure. In other words, it is the quantity that measures the number of unit squares that cover the surface of a closed figure. The standard unit of area is square units which is generally represented as square inches, square feet, etc.
First of all, lets calculate the points of intersection (P, Q, R)
x²+3=(x+2) +5
x²-2=√(x+2)
x⁴+4-4x²=x+2
x⁴-4x²-x+2=0
(x-2)(x³+2x²-1)=0
(x-2)(x+1)(x²+x-1)=0
x=2, -1, -1±√(1+4)/2
Clearly, the x-coordinate of Q is -1, P is -1-√5/2, R is -1+√5/2, S is 2
So the limit of integration will be
P( (-1-√5)/2, (-1-√5/2)² +3)=P((-1-√5)/2, (3+√5/2))
Q(-1, (-1)²+3)=Q(-1, 4)
Area A:
\(\int\limits^\frac{3+\sqrt{5} }{2} _4 {-\sqrt{-y-3}-((y-5)^2 -2)} \, dx\)
= \([\frac{-(y-3)^\frac{3}{2} }{\frac{3}{2} }-\frac{(y-5)^3}{3}+3y]^{\frac{3+\sqrt{5} }{2} }_4\)
= 2.07
Area B:
\(\int\limits^\frac{-1+\sqrt{5} }{2} _4 {-\sqrt{x+2}+5-(x^2+3)} \, dx\)
= \([\frac{-(x+2)^\frac{3}{2} }{\frac{3}{2} }+5x-\frac{x^3}{3}-3x]^{\frac{-1+\sqrt{5} }{2} }_{-1}\)
= 0.667
Total area = 2.07+0.667
= 2.737
Therefore, the total area from the graph is 2.737.
Learn more about the area here:
https://brainly.com/question/27683633.
#SPJ1
4. The length of a rectangle is eight more
than twice its width. The perimeter is 96
feet. Find the dimensions of the rectangle.
Variable:
Equation:
Solution:
Answer:
L = length
W = width
L = 2(2W + 8)
W = 2W
Perimeter: 96 = 4W + 16 + 2W
96 = 6W + 16
80 = 6W
13.33 = W
Plug this into the equation:
2(13.33) + 8
26.67 + 8 or 34.67 = L
Step-by-step explanation:
You have a huge review project for English where you have to answer 300 questions. You start on it after
school at 5:00. At 6:30, you have answered 55 questions.
Write a linear equation to represent Q questions remaining after h hours working on the homework.
write a linear equation to represent Q for questions remaining after h for hours working on the homework
Answer:
This blog entry describes what my students and I did during Week 2 of the Emerging Tech (NSF Grant) project. The events in this blog entry took place at the same time as the events in this article. As a pair, these describe what a PBL teacher does while running two projects in two different preps at one time. To see accounts on earlier or later weeks of these projects, go here.
Please answer this it is due in 10 mins!! Will give brainliest <3
Answer:
she got: $117.25
Step-by-step explanation:
Total money she got after selling:
\(8*5.75+10*6.25+5*\frac{2.5}{2} +2*\frac{1}{3} * 3.75\)
$117.25
Answer:
113.5 i hope its right
Step-by-step explanation:
plz give brainliest
Suppose that Motorola uses the normal distribution to determine the probability of defects and the number of defects in a particular production process. Assume that the production process manufactures items with a mean weight of 10 ounces. Calculate the probability of a defect and the suspected number of defects for a 1,000-unit production run in the following situations.
(a) The process standard deviation is 0.25, and the process control is set at plus or minus one standard deviation. Units with weights less than 9.75 or greater than 10.25 ounces will be classified as defects. If required, round your answer for the probability of a defect to four decimal places and for the number of defects to the nearest whole number.
Probability of a defect:
Number of defects:
The probability of a defect is 5.7330 x \(10^{-5}\) and the number of defects is 5.73.
To calculate the probability of a defect, we need to find the area under the standard normal curve that lies outside of the process control limits of 9.75 ounces and 10.25 ounces. We can use the standard normal distribution table to find this area.
First, we need to standardize the weight limits as follows -
\(Z_{lower}\) = (9.75 - 10) / 0.25 = -4
\(Z_{upper}\) = (10.25 - 10) / 0.25 = 4
Next, we will find the area under the standard normal curve that lies outside of these limits as follows -
P(Defect) = P(Z < -4) + P(Z > 4)
Using a standard normal distribution table, we can find that P(Z < -4) = 2.8665 x \(10^{-5}\) and P(Z > 4) = 2.8665 x \(10^{-5}\) .
So, the total probability of a defect is -
P(Defect) = 2.8665 x \(10^{-5}\) + 2.8665 x \(10^{-5}\) = 5.7330 x \(10^{-5}\)
Finally, we will find the number of defects for a 1,000-unit production run as follows -
The number of defects = 1000 * 5.7330 x \(10^{-5}\) = 5.73 (rounded to the nearest whole number).
Read more about Probability:
brainly.com/question/24756209
#SPJ4
Please help with the problem
Explanation:
If you are dividing a fraction, the divisor will become it's reciprocal and the division sign will change to a multiplication sign.
Solution (a):
3 ÷ 1/4 = 12=> 3 x 4 = 12=> 12 = 12 [True]Solution (b):
7 ÷ 1/2 = 1/14=> 7 x 2 = 1/14=> 14 ≠ 1/14 [False]Solution (c):
1/5 ÷ 4 = 20=> 1/5 x 1/4 = 20=> 1/20 ≠ 20 [False]Solution (d):
1/2 ÷ 5 = 5/2=> 1/2 x 1/5 = 5/2=> 1/10 ≠ 5/2 [False]Solution (e):
1/7 ÷ 3 = 1/21=> 1/7 x 1/3 = 1/21=> 1/21 = 1/21 [True]Final Answer(s)
Option AOption E1. Find the conditional probability of the indicated event when two fair dice (one red and one green) are rolled. The sum is 6, given that the green one is either 4 or 1.
2. Find the conditional probability of the indicated event when two fair dice (one red and one green) are rolled. The red one is 6, given that the sum is 11.
Answer:
1. 1/6
2. 1/6
Step-by-step explanation:
Let A be the event that the sum of the two die is 6 and B be an event that the green die is either 4 or 1.
The conditional probability will be given by P (A/B) = P (A∩B)/ P (B).
Now the total sample space consists of 36 outcomes .
And to find (A∩B) we need to find the outcomes in which green die is either 4 or 1 and the sum of the two die is 6.
So when green is 1 red must be 5
So when green is 4 red must be 2
So there are two ways in which green die is either 4 or 1 and the sum of the two die is 6.
Therefore the probability of (A∩B)= P (A∩B)= 2/36= 1/18
Now we find the probability of green die having 4 or 1
So when green is 4 red can have all the numbers from 1- 6
And when green is 1 red can have all the numbers from 1- 6
The total number would be 12 .
So probability of green die having 1 or 4 is given by = P (B)= 12/36
Now the conditional probability = P (A/B) = P (A∩B)/ P (B)=1/18/ 1/3
= 3/18= 1/6
2. Similarly we find the conditional probability of the two die when the red one is 6, given that the sum is 11.
When red is 6 the green must be 5 to get 11. So the probability
=P (A∩B)= 1/36
Now we find the probability of red die having 6 =P(B)= 6/36
Now the conditional probability = P (A∩B)/P(B) = 1/36/ 6/36= 1/6
Answer 1:
Let A be the event that the sum of the two die is 6 Let B be an event that the green die is either 4 or 1.
Conditional probability Formula :
P (A/B) = P (A∩B)/ P (B).
Total sample space=36 outcomes
Conditions are :
So when green is 1 red must be 5 So when green is 4 red must be 2 So there are two ways in which green die is either 4 or 1 and the sum of the two die is 6.
The probability of (A∩B)= P (A∩B)= 2/36= 1/18
Now we find the probability of green die having 4 or 1
When green is 4 red can have all the numbers from 1- 6
And when green is 1 red can have all the numbers from 1- 6
Total number = 12
P (B)= 12/36
Therefore, conditional probability = P (A/B)
P (A/B) = P (A∩B)/ P (B) P (A/B)=1/18/ 1/3 P (A/B)= 3/18 P (A/B)= 1/6
The conditional probability of the indicated event when two fair dice are rolled will be 1/6.
Answer 2:
Let A be the event that the sum of the two die is 6 Let B be an event that the green die is either 4 or 1. The sum is 11.
Condition :
When red is 6 the green must be 5 to get 11.
P (A∩B)= 1/36
The probability of red die having 6 =P(B)= 6/36
The conditional probability= P (A∩B)/P(B)
P (A∩B)/P(B) = 1/36/ 6/36P (A∩B)/P(B)= 1/6The conditional probability of the indicated event when two fair dice are 1/6.
Learn more :
https://brainly.com/question/14660973?referrer=searchResults
Alene wants to buy a pencil pouch that costs $16.00 . She must also pay 5 % sales taxIf she pays with a $20 bill how much change will she receive back?
Explanation:
First we have to find the total cost of the pencil pouch including taxes:
\(\text{tax}=16\cdot\frac{5}{100}=16\cdot0.05=0.8\)The tax is $0.80, so the total cost of the pencil pouch is $16.80.
The change is the difference between how much Alene pays and how much does the item cost:
\(20-16.80=3.20\)Answer:
She will receive $3.20 back
3 72/120 rounded to the nearest half
Answer:
4
Step-by-step explanation:
3 72/120
÷24 ÷24
3 3 / 5 = 3 + 3/5 = 3 + 6/10 = 3 + 0.6 = 3.6 ≈ 4 rounded to the nearest half since 3.6 is closer to 4 than 3
There are 7 black balls and 10 red balls in an urn. If 4 balls are drawn without replacement, what is the probability that at least 3 black balls are drawn
Answer:
1/16? I say this cause uh it seems like a rational number
Explain what needs to be fixed.
2n + 10 = -2
2n = 12 Step one
n = 6 Step two
Answer:
step one needs to be fixed... you are supposed to subtract 10 from both sides of the equation.
Step-by-step explanation:
STEP 1: Move all terms not containing n to the right side of the equation.
\(2n=-12\)
STEP 2: Divide each term by 2 and simplify.
\(n=-6\)
total area of 3 figures
Given dimensions:
x=31 feet, y= 24 feet and z= 95 feet
We can split the shape above into 3 components: A, B, and C
To find the total area, we will find the sum of the areas of each component
For A
The shape of A is that of a semi-circle.
The area of a semi-circle is given to be
\(\text{Area}=\text{ }\frac{1}{2}\text{ x }\pi r^2\)The radius will be the diameter divided by 2
y= diameter
r= radius = y/2
r=24/2 =12 feet
pi=3.14
\(\begin{gathered} \text{Area}=\frac{1}{2}\text{ x 3.14 x 12 x 12} \\ \text{Area}=226.08\text{ ft}^2 \end{gathered}\)For B
The shape is a rectangle
The area of a rectangle is given by
A = l x b
where l = 31 and b = 24
Area = 31 x 24
Area = 744 square feet
For C
The shape is a triangle
The area of the triangle is given by
\(A=\frac{1}{2}\text{ x base x height}\)base = 64 feet, height = 24 feet
\(\text{Area}=\frac{1}{2}\text{ x }64\text{ x 24 =768 ft}^2\)The total area is
22
Justin owns a small business selling used books. He knows that in the last week 76 customers paid cash, 4 customers used a debit card, and 42 customers used a credit card. If next week, he is expecting 200 customers, about how many would you expect to pay with cash? Round your answer to the nearest whole number.
Rounding this to the nearest whole number, we can estimate that about 125 customers will pay with cash next week.
Based on the previous week's data, the percentage of customers who paid with cash is (76/122)*100 = 62.3%. If we assume that this percentage will remain constant for the next week, then we can estimate that out of 200 customers, approximately 62.3% will pay with cash.
To calculate this, we can use the proportion:
(76/122) = (x/200)
where x is the number of customers we expect to pay with cash next week.
Solving for x, we get:
x = (76/122)*200 = 124.59
Rounding this to the nearest whole number, we can estimate that about 125 customers will pay with cash next week.
For more questions on cash
https://brainly.com/question/14153580
#SPJ11
Oreana's 150 g bag of trail mix is x% raisins. Brandon's 250 g bag of trail mix is y% raisins. They combine the two mixes together in one bowl.
Write an expression which shows how many grams of raisins are in the bowl
The expression which shows how many grams of raisins are in the bowl will be
(0.01x) * 150 + (0.01y) * 250
What is the expression that shows the raisin?An expression is simply used to show the relationship between the variables that are provided or the data given regarding an information. In this case, it is vital to note that they have at least two terms which have to be related by through an operator. Some of the mathematical operations that are illustrated in this case include addition, subtraction, etc.
The expression which shows how many grams of raisins are in the bowl will be:
= (x% × 150) + (y% × 250)
= (0.01x) * 150 + (0.01y) * 250
This equation represents the total grams of raisins in the bowl by multiplying the weight of the Oreana's bag by the percentage of raisins in it, and adding that to the product of the weight of Brandon's bag and the percentage of raisins in it. The "0.01" is used to convert the percentages from x and y to decimal form.
Learn more about expressions on:
https://brainly.com/question/723406
#SPJ1
What are the coordinates of the graphed point
Answer:
(1, -4.5)
4th quadrant has positive x's and negative y's
- Which function is graphed below?
Answer:
√2+x
Step-by-step explanation:
-
Answer:
I'm pretty sure the answer is the function starting with g(x); its a transformation going left 2 units, meaning that you add two to \(\sqrt{x}\). Think off negative as positive and positive as negative.
Step-by-step explanation:
Sorry I'm late! hopefully this helps others as well :)
HEELLLLLPPPPPPPP PLLLSSSS THIS ISS TIMMMEEEEDDDDD
Answer:
60°
Step-by-step explanation:
Assuming EAD is a straight line: 180 - 100 - 20 = 60°
6x - 2y = 18
3x + 4y= -6
Which of the following ordered pairs (x, y) is the
solution to the system of equations shown above?
A. (-3,2)
B. (-2,3)
C. (2, -3)
D. (3,-2)
Answer:
C. (2, -3)
Step-by-step explanation:
Easiest and fastest way to get the solution set is to graph the systems of equations and analyze where the graphs intersect.
A $5000 investment pays 3% interest compounded monthly. To the nearest cent, what will be the value of the investment after 10 years?
Answer:
We can use the formula for compound interest to solve this problem. The formula is:
A = P * (1 + r/n)^(n*t)
where:
A = the amount of money after t years
P = the principal amount (the initial investment)
r = the annual interest rate (as a decimal)
n = the number of times the interest is compounded per year
t = the number of years
In this case, P = $5000, r = 0.03, n = 12 (since the interest is compounded monthly), and t = 10. Plugging these values into the formula, we get:
A = 5000 * (1 + 0.03/12)^(12*10)
A = 5000 * (1.0025)^120
A ≈ $6,621.36
So the investment will be worth approximately $6,621.36 after 10 years.
16 families went on a trip which cost them Rs 2,16,352. How much did each
family pay?
Given that 16 families went on a trip and the cost of the trip was Rs. 2,16,352.The amount paid by each family is to be determined by unitary method Hence each family paid Rs.13522
Now, let's solve this by using the method of unitary method. To find the cost of 1 family trip, we will divide the total cost of the trip by the number of families.2,16,352 / 16 = 13,522 So, the cost of the trip per family is Rs. 13,522.Hence, each family paid Rs. 13,522 for the trip.
to know more about unitary method
https://brainly.com/question/28276953
Answer:
Step-by-step explanation
1. The total cost of the trip for all 16 families is Rs 2,16,352.
2. To find out how much each family paid, we need to divide the total cost by the number of families: Rs 2,16,352 ÷ 16.
3. When we do the division, we get the result: Rs 13,522.
Now let's check if this result is correct:
1. If each family paid Rs 13,522 for the trip, then the total cost for all 16 families would be: 16 × Rs 13,522 = Rs 2,16,352.
2. This is exactly the same as the total cost given in the problem statement.
So we have shown that each family paid **Rs 13,522** for the trip
What is the length of the two equal sides?
What is the length of the third side?
Answer:
Two equal sides: 15 centimeters, Third side: 21 centimeters
Step-by-step explanation:
Sorry If I'm late, so how I solved this was by using variables and algebra. The perimeter of a triangle is the sum of all three sides. Let's now break down the problem and use the useful information, we know that the perimeter is 51 centimeters, and that there are two equal sides and the third side is 6 inches longer than the two equal sides. So it is pretty much an isosceles triangle since it has two sides of equal length and the third side is not equal to the others. So we can make an equation with variables. 51(Perimeter)=x(first equal side)+x(second equal side)+x(third side)+6(The third side is 6 centimeters longer than the two other sides), now we simplify the equation just like we do with most algebraic equations. x+x+x can be turned into 3x. This now makes the equation 51=3x+6, we can subtract 6 from both sides to just have the variable on one side, 3x+6-6, both sixes cross out making it 3x, 51-6 is equal to 45. Now we divide by three on both sides to give just x and find out what x is. 3x/3=x and 45/3=15. x=15, x is the equal side length, so the two equal sides are 15 centimeters and the third one is 21 centimeters. The reason for this is because 15+6=21. I hope this helped with your learning, if I am wrong please tell me :)
52. Find a vector v whose magnitude is 3 and whose component in the i direction is equal to the component in the j direction.
The vector whose magnitude is 3 and whose components I. the I and j direction are equal is; <3√2/2i, 3√2/2j>.
Which vector is as described in the task content above?It follows that the magnitude of a vector in terms of its components in the i and j direction is;
M = √(x² + y²).
On this note, since the i and j components are equal; x = y and hence, we have;
3 = √(x² + x²).
3² = 2x²
x² = 9/2
x = 3/√2
x = 3√2/2
On this note, the required vector which is as described is; <3√2/2i, 3√2/2j>.
Read more on vector components;
https://brainly.com/question/26187714
#SPJ1
By the Remainder Theorem, what can be said about the polynomial function w(x) if w(−5)=3 ?
the remainder must be _________ when w(x) is divided by _________
thanks so much!
well, ding ding ding!! let's recall the remainder's theorem
if some function f(x) has a factor of say x - a, then if we plug the "a" in f(x) what we get is the remainder, that is, the remainder in a division of f(x) by (x-a).
all that mouthful said, well, since we know of w(-5), that means w(x) must have a factor of x - (-5) or namely x + 5.
we also know that w(-5) = 3, well, that means that 3 is the remainder of such division, that is, w(x) ÷ (x + 5), gives us a remainder of 3.
What is slope I need help finding the slope please help me
The slope is 3 this is intergers solution the right side is positive and left side is negative